Questions tagged [sequences-and-series]

for questions about sequences and series, e.g. convergence, closed form expressions, etc. Note that there is a different tag for spectral sequences, and also note that MathOverflow is not for homework. Please consider consulting the online encyclopedia for integer sequences, if you are trying to identify a given sequence that you have found in your research.

Filter by
Sorted by
Tagged with
15 votes
1 answer
1k views

The Mompox Sequence: are all its terms different?

The Mompox Sequence, $a(n):=1, 2, 6, 24, 120, 20, \ldots$ (OEIS A008336), is the sequence of positive integers whose first term is $1$, and in which the $n$-th term (after the first one) equals the ...
Bernardo Recamán Santos's user avatar
6 votes
2 answers
1k views

5n+1 sequence starting at 7

Consider the following variant of the Collatz function: $f:\mathbb N\rightarrow\mathbb N$ is defined by \begin{equation} f(n):=\begin{cases} n/2 & \text{if $n$ is even}\\ 5n+1 & \...
Riemann's user avatar
  • 537
10 votes
2 answers
1k views

Does a conditionally convergent sum with random signs converge almost surely?

Let $\sum a_n$ be a conditionally convergent sum of real numbers, and $\epsilon_n$ a sequence of independent identically distributed Bernoulli random variables with $\epsilon_n = 1$ or $-1$ with ...
Nate River's user avatar
  • 4,832
8 votes
1 answer
1k views

When can a sum be re-signed to converge to any limit?

Let $a_n$ be a sequence of positive real numbers with $\sum a_n < \infty$. What are the necessary and sufficient conditions for the following to hold? For any $S \in \mathbb R$ with $-\sum a_n \...
Nate River's user avatar
  • 4,832
2 votes
1 answer
145 views

Growth rate of elementary sequences

We consider three sequences $(x_n),(y_n),(z_n)$, where $(x_n) \in \ell^1$ is positive and the other two sequences are merely assumed to be positive, i.e. $y_n,z_n \ge 0$ where $0<z_n<z_{n+1}$ is ...
António Borges Santos's user avatar
4 votes
1 answer
279 views

3 divides coefficents of this $q$-series

Denote $\phi(q):=\prod_{j\geq1}(1-q^j)$ and let $\xi=e^{\frac{2\pi i}3}$ be a cube root of unity. Define the sequence $u(n)$ by $$\prod_{n\geq1}\prod_{s=1}^2(1-q^n\xi^{ns})(1-q^{2n}\xi^{ns}) =\sum_{n\...
T. Amdeberhan's user avatar
2 votes
5 answers
909 views

Binomial series

I am interested in the limit $\frac{\sum_{k=0}^n \sqrt{k}\cdot\binom{n}{k}}{\sqrt{n}\cdot2^n}$ as $n$ goes to infinity. Any reference or argument? In general what do we know about the asymptotic ...
Morteza's user avatar
  • 628
3 votes
1 answer
375 views

Generalized harmonic numbers and Riemann zeta function

The $n$-th harmonic number is defined as $$ H_{n}=\sum\limits_{k=1}^{n}\frac{1}{k}, $$ and the generalized harmonic numbers are defined by $$ H_{n}^{(m)}=\sum\limits_{k=1}^{n}\frac{1}{k^m}. $$ It is ...
Notamathematician's user avatar
3 votes
1 answer
287 views

Smallest number of subsets whose squares cover the whole square

Let $2 \leq k \leq n$ be integers, let $[n] := \{1,2,\ldots,n\}$, and for a subset $A \subseteq [n]$ let $A^2 := A \times A$ be the Cartesian product of $A$ with itself and let $|A|$ denote the ...
Nathaniel Johnston's user avatar
1 vote
1 answer
123 views

Is a bounded convex function $g$ that is non-negative on this particular convex set also non-negative on the its closure?

Setup : Let $S$ be the simplex on $\mathbb N$, i.e. the set of probability distribution on the natural numbers. Suppose we have $p\in S$ such that, for all $n\geq 1$, $p_n > 0$. For any $\emptyset\...
P. Quinton's user avatar
1 vote
1 answer
85 views

$R$-recursion for the A307389

Let $a(n)$ be A307389 i.e. an integer sequence such that its exponential generating function $A(x)$ satisfies $$ A(x)=\exp\left(\frac{\exp(2x)-2\exp(x)+2x+1}{2}\right) $$ The sequence begins with $$ 1,...
Notamathematician's user avatar
43 votes
3 answers
2k views

Proving $\sum_{i=1}^{n}\sum_{j=1}^{n}\left\{\frac{x_{i}}{x_{j}}\right\}\le \frac{9}{14}n^2$?

For any postive integer $n$ and for any postive real numbers $x_{1},x_{2},\cdots,x_{n}$, show that $$\sum_{i=1}^{n}\sum_{j=1}^{n}\left\{\dfrac{x_{i}}{x_{j}}\right\}\le \dfrac{9}{14}n^2$$ Let \begin{...
math110's user avatar
  • 4,230
1 vote
1 answer
111 views

Properties of the relatively bounded probability distributions on the simplex over the natural numbers

Setup : Let $S$ be the simplex over $\mathbb N$, i.e. the set of probability distribution over $\mathbb N$. Let $p\in S$ be such that $0 < p_n$ for all $n\in \mathbb N$. Let $H$ be the Hilbert ...
P. Quinton's user avatar
1 vote
0 answers
71 views

Slightly modified program for the A345253 such that specific partial sums equal A066258

Let $F(n)$ be A000045 i.e. Fibonacci numbers. Here $$ F(n) = F(n-1) + F(n-2), \\ F(0) = 0, F(1) = 1 $$ Let $a(n)$ be A345253 i.e. maximal Fibonacci tree: arrangement of the positive integers as ...
Notamathematician's user avatar
6 votes
1 answer
311 views

A conjectured series expression for the Riemann $\xi$-function and/or Completed L-series. Could this be proven?

This post builds on an MSE question about a conjectured series expression for the Riemann $\xi$-function: $$\xi(s) = \xi(1-s) = \sum_{n=1}^\infty (-1)^{n+1}\,\big(\xi\left(s+in\right)+\xi\left(1-s+in\...
Agno's user avatar
  • 4,169
4 votes
1 answer
192 views

Partition numbers as the specific sums of the A161511

Let $p(n)$ be A000041 i.e. number of partitions of $n$ (the partition numbers). Let $$ \ell(n)=\left\lfloor\log_2 n\right\rfloor $$ Let $a(n)$ be A161511 i.e. number of $1\cdots0$ pairs in the ...
Notamathematician's user avatar
1 vote
0 answers
168 views

Amateur Exploring the 'Honeycomb Sequence': A Novel Mathematical Pattern Derived from Pascal's Triangle [closed]

I am an amateur, and for fun, I was studying a specific number sequence I called the "Honeycomb Sequence," derived from hexagonal patterns in Pascal's Triangle. The sequence involves ...
thomasfreund's user avatar
0 votes
1 answer
96 views

Finding the smallest possible value of $|S_n|$ for Sequence and Series in real analysis [closed]

Let $𝑋_1,𝑋_2, … , 𝑋_𝑛$ be a sequence of independent random variables with $𝑃(𝑋_𝑛 = 4^𝑛) = 𝑃(𝑋_𝑛 = −4^𝑛) = \frac12$. Let $𝑆_𝑛 = 𝑋_1 + 𝑋_2 + ⋯ + 𝑋_𝑛$. If $A_n=\sup\, \{𝑟 ∈ \Bbb R: 𝑃(|...
john22445's user avatar
2 votes
2 answers
247 views

If $\inf\{b\in\mathbb{R}\mid\sum_{n=1}^{\infty}e^{-ax_n-by_n}<+\infty\}=1-a$ for all $a\in [0,1]$, does this equality hold for all $a\in\mathbb{R}$?

Let $\left\{x_n\right\}_{n=1}^{+\infty},\left\{y_n\right\}_{n=1}^{+\infty}\subset [0,+\infty)$ be two sequences of non-negative real numbers. Suppose there exist $\lambda\ge 1, c\ge 0$ such that $\...
YC Su's user avatar
  • 23
6 votes
2 answers
709 views

Recreation with Catalan

Consider the well-known sequence $C_k=\frac1{k+1}\binom{2k}k$ of Catalan numbers. I came across the below identity while working with certain generating functions. I thought it might be of interest to ...
T. Amdeberhan's user avatar
1 vote
0 answers
222 views

Is $\sum\limits_{k=1}^nk^m=S_3(n)\cdot\dfrac{P_{m-3}(n)}{N_m}$ for odd $m>1,\sum\limits_{k=1}^nk^m=S_2(n)\cdot\dfrac{P_{m-2}'(n)}{N_m}$ for even $m$?

I asked this question here When I was in high school, I was fascinated by $$ \sum\limits_{k=1}^n k= \frac{n(n+1)}{2} $$ so I tried to find the general value of the sum $$ \sum\limits_{k=1}^n k^m\;\...
pie's user avatar
  • 207
3 votes
0 answers
69 views

$R$-recursion for the A249833 (similar to A235129)

Let $a(n)$ be A249833 i.e. an integer sequence such that its exponential generating function $A(x)$ satisfies $$ A(x) = 1 + \int A(x) + (A(x))^2\log A(x)\,dx $$ The sequence begins with $$ 1, 1, 2, 7, ...
Notamathematician's user avatar
0 votes
1 answer
97 views

The sequence has a stationary accumulation point

Let $f:\mathbb{R}^n\rightarrow \mathbb{R}$ be a smooth (continuously differentiable), convex function with a non-empty set of minimizers and $\{x^k\}$ be a sequence such that (a) $\{x^k\}$ has an ...
Dat Ba Tran's user avatar
2 votes
0 answers
102 views

$R$-recursion for the A235129

Let $a(n)$ be A235129 i.e. an integer sequence such that its exponential generating function $A(x)$ satisfies $$ A'(x) = 1 + A(x)\exp(A(x)) $$ The sequence begins with $$ 1, 1, 3, 12, 64, 424, 3358, ...
Notamathematician's user avatar
2 votes
1 answer
280 views

Are these conditions regarding products of consecutive terms in a sequence of positive numbers equivalent?

Assume $w_n$ is a bounded (weight) sequence of positive numbers. We want to consider products of consecutive terms in this sequence. For $i,j\in \mathbb{N}$, define $M_i^j = w_i w_{i+1}\cdots w_{i+j-1}...
David Walmsley's user avatar
5 votes
1 answer
248 views

Precise asymptotic estimate of a recurrence sequence involving a square root

Consider a recurrence sequence defined like this: $$ \begin{cases} x_0 = \varepsilon \\ x_{n+1} = x_n + \varepsilon \sqrt{x_n}. \end{cases}$$ I am interested in estimating the value of $x_{\...
tommy1996q's user avatar
5 votes
1 answer
366 views

Closed-form for the number of partitions of $n$ avoiding the partition $(4,3,1)$

Let $a(n)$ be A309099 i.e. the number of partitions of $n$ avoiding the partition $(4,3,1)$. We say a partition $\alpha$ contains $\mu$ provided that one can delete rows and columns from (the Ferrers ...
Notamathematician's user avatar
2 votes
0 answers
62 views

Elementary recursion for the A258173

Let $a(n)$ be A258173 i.e. sum over all Dyck paths of semilength $n$ of products over all peaks $p$ of $y_p$, where $y_p$ is the $y$-coordinate of peak $p$. A Dyck path of semilength $n$ is a $(x,y)$-...
Notamathematician's user avatar
27 votes
5 answers
3k views

How to show a function converges to 1

Consider the following recurrence relation in two variables: $$f(a, b) = \frac{a}{a+b} f(a-1,b)+ \frac{b}{a+b}f(a+1,b-1) $$ for positive integers $a$ and $b$, with the boundary conditions $f(0,b)=0$ ...
Simd's user avatar
  • 3,195
1 vote
1 answer
81 views

Formulas for partial composed product

Let $A(x) = \prod\limits_i (x-\lambda_i)$ and $B(x) = \prod\limits_j (x-\mu_j)$. Then, their composed product is defined as $$ (A*B)(x) = \prod\limits_{i,j} (x-\lambda_i \mu_j). $$ Generally, we can ...
Oleksandr  Kulkov's user avatar
0 votes
0 answers
174 views

Mean Cauchy sequences

Let $X$ be a $\sigma$-finite measure space. A sequence of functions $f_n \in L^1 (X)$ is said to be Cauchy in mean if $$\lim_{K \to \infty} \limsup_{N, M \to \infty} \frac{1}{NM} \sum_{i = K+1}^{K + N}...
Nate River's user avatar
  • 4,832
3 votes
2 answers
240 views

Abel–Plana formula with fractional offset

The Abel–Plana formula compares the sum $\sum_{n=0}^\infty f(n)$ to the integral $\int_0^\infty f(x)\,dx$, \begin{equation} \sum_{n=0}^{\infty}f\left(n\right)-\int_{0}^{\infty}f\left(x\right)dx=\frac{...
Carlo Beenakker's user avatar
4 votes
1 answer
383 views

Series convergence if $\sum a_n^2 < \infty$

There are quite a few simple results about convergent/divergent series derived from similar ones. Here is a question in the same spirit that I saw posted on another forum. Unfortunately, I don't have ...
Ivan's user avatar
  • 689
6 votes
1 answer
374 views

How to show that $\log 2(1/2\log 2\log 4 + 1/3\log 3\log 6 + \dotsb) + 1/2\log 2 - 1/3\log 3 + 1/4\log 4 - \dotsb = 1/\log 2$ [closed]

I've been studying Ramanujan's work and I stumbled upon this question in the book: Collected Papers of Srinivasa Ramanujan. In there I found question number 769 which is about an infinite sum with ...
Euler-Masceroni's user avatar
2 votes
1 answer
97 views

Natural density of thickly syndetic set

A syndetic set $S$ is a subset of the natural numbers $\mathbb{N}$ or integers $\mathbb{Z}$, having the property of "bounded gaps": that the sizes of the gaps in the sequence of natural ...
Matej Moravik's user avatar
1 vote
1 answer
163 views

Maximization of $\ell^2$-norm

Consider for $r,c>0$ the set $$X_{r,c}=\{x \in \ell^1(\mathbb{N}) \mid \|x\|_1=r,\, \forall i \in \mathbb{N}: |x_i|<c\}.$$ Then I can show that $\inf_{x \in X_{r,c}} \|x\|_2 = 0.$ But is it ...
SequenceGuy's user avatar
2 votes
1 answer
170 views

Does every real number $r\in [0,1]$ have a rational sequence $q_n\to r$ s.t. $q_n$ has (simplified) denominator $n$? [closed]

This seems pretty trivial but I can't seem to figure it out. I think it's obviously true, given an unconstrained convergent sequence we just have to add some filler elements, but I'm having trouble ...
uniform_on_compacts's user avatar
14 votes
0 answers
376 views

Will a unit disk be completely covered by randomly placed disks of area $\pi,\frac{\pi}{2},\frac{\pi}{3},\dots$ with probability $1$?

On a "bottom" disk of area $\pi$, we place "top" disks of area $\pi,\frac{\pi}{2},\frac{\pi}{3},\dots$ such that the centre of each top disk is an independent uniformly random ...
Dan's user avatar
  • 2,653
3 votes
0 answers
125 views

Series acceleration for $\sum_{k=0}^\infty\left(\frac{H^k}{k!}\right)^\beta$, $\beta\ll 1$

The probability mass of the Conway-Maxwell-Poisson variable $K$ is given by $$ \mathsf P(K=k)=\frac{1}{Z(H,\beta)}\left(\frac{H^k}{k!}\right)^\beta $$ where $$ Z(H,\beta)=\sum_{k=0}^\infty\left(\frac{...
Aaron Hendrickson's user avatar
0 votes
1 answer
288 views

Uniqueness of the $J$ invariant

It seems that The $J$ invariant is the unique modular function of weight zero for $\operatorname{SL}(2,\mathbb{Z})$ which is holomorphic away from a simple pole at the cusp such that $$J(e^{2\pi i/3})...
Nomas2's user avatar
  • 357
5 votes
1 answer
465 views

How can I evaluate the following sum?

While studying sequences and series, I came across summations of geometric series. I am able to derive the sum of a geometric progression and that of arithmetico–geometric sequence. But taking a step ...
RajaKrishnappa's user avatar
2 votes
0 answers
164 views

How to prove this weighted sum inequality with non-increasing sequences?

Problem I have two non-increasing sequences, $X = (x_1, x_3, x_5, \ldots, x_{n-1})$ and $Y = (y_1, y_3, y_5, \ldots, y_{n -1})$, $n$ is an even integer. I want to prove this inequality: $$ \sum_{i=1}^{...
birdlpy's user avatar
  • 29
6 votes
1 answer
311 views

Convergence of derived series

There are quite a few simple results about convergent/divergent series derived from similar ones. So, here is something that I came across recently: Let $\sum_{n=1}^\infty a_n$ consist of positive ...
Ivan's user avatar
  • 689
0 votes
1 answer
245 views

Nature of $ \sum_{n \geq 1} \frac{ \cos(n) \sin(n+1) }{n} $ [closed]

I'm trying to determine the nature of this series $ \sum_{n \geq 1} \frac{ \cos(n) \sin(n+1) }{n} $, but I'm not getting anywhere. I've tried using the Abel and trigonometric formulas, but I can't ...
user avatar
0 votes
0 answers
138 views

Dark side of the self-inverse permutation

Let $$ \ell(n) = \left\lfloor\log_2 n\right\rfloor $$ Let $$ f(n) = 2^{\ell(n)} $$ Let $p_1(n)$ be an arbitrary self-inverse permutation of the non-negative integers such that $p_1(n)<2^k$ iff $n&...
Notamathematician's user avatar
3 votes
4 answers
445 views

Asymptotic for Ramanujan's $\tau$-function

The Ramanujan's $\tau$-function is defined by $$q\prod_{n=1}^\infty (1-q^n)^{24}=\sum_{n=1}^\infty \tau (n)q^n$$ where $|q|\lt 1$. Is there a known asymptotic formula for $\tau (n)$ or $|\tau (n)|$, i....
Nomas2's user avatar
  • 357
8 votes
2 answers
391 views

Let $(a_n)_{n\in N}=(1,2,3,4,6,8,9,12,\cdots)$ list the set$\{2^n3^m\mid m,n\in N\}$. Find $α$ such that $(a_n)\alpha\pmod1$ is not equidistributed

Let $$(a_n)_{n \in \mathbb{N}} = (1,2,3,4,6,8,9,12,16,18,\cdots)$$ be a sequence that is a listing of the set $$\{2^n3^m \mid m,n \in \mathbb{N}\}$$ We need to find an irrational number $\alpha$ such ...
Miranda's user avatar
  • 183
2 votes
0 answers
124 views

Limit of scaled infinite sum with Dirichlet characters modulo 4: is it zero?

I am trying to get an asymptotic formula such as $$ L_4(s, n) \sim L_4(s) + \rho_n(s)\Lambda_n + \frac{\alpha(s)}{\sqrt{n}} + \frac{\beta(s)}{\sqrt{n\log n}}+\cdots$$ where $L_4(s, n)$ is the first $n$...
Vincent Granville's user avatar
1 vote
1 answer
228 views

A vanishing sum and related $p$-adic congruences

Recently I had a curious discovery. Namely, I have made the following conjectures. Conjecture 1. We have the identity $$\sum_{k=0}^\infty\frac{(10k-1)\binom{3k}k\binom{6k}{3k}}{(2k+1)512^k}=0.\label{1}...
Zhi-Wei Sun's user avatar
  • 14.5k
0 votes
0 answers
53 views

Stolarsky representation from Zeckendorf representation with some pairs of bits inverted

Let $a(n)$ be A200714 i.e. Stolarsky representation interpreted as binary to decimal integers. Let $b(n)$ be A003714 i.e. Fibbinary numbers (Zeckendorf representation interpreted as binary to decimal ...
Notamathematician's user avatar

1
2
3 4 5
35